LSAT and Law School Admissions Forum

Get expert LSAT preparation and law school admissions advice from PowerScore Test Preparation.

User avatar
 appletree
  • Posts: 13
  • Joined: Feb 11, 2021
|
#87446
Hello, could someone please explain why choice A is incorrect?
The way I reasoned it was that we are told that class divisions strengthen divisive political factions (discord).
So then the discord would lead to even greater class divisions, which would then reinforce the discord (so the cycle continually reinforces itself).
Therefore, I thought this choice would strengthen the reasoning by showing why economic expansion is a good idea.
 Adam Tyson
PowerScore Staff
  • PowerScore Staff
  • Posts: 5153
  • Joined: Apr 14, 2011
|
#87865
You're right that this answer sets up a vicious circle, appletree, with discord contributing to income inequity, which makes more discord, etc. But the problem with this answer is that it doesn't add anything to the conclusion that democracies should try to have constantly expanding economies, because it doesn't tell us that those policies will do anything to change the income inequities. What if a constant expansion of the economy only benefited the top 1%, and made income inequity even worse?

We need an answer that shows a connection between constant expansion and greater mobility between those economic strata. Answer A doesn't make that connection for us. The rich might just continue getting richer, continuing the cycle!
 ltowns1
  • Posts: 61
  • Joined: May 16, 2017
|
#89530
I circled (C) to begin with, but went with (E) instead. I felt (C) was too weak of an answer. I was looking for something more like it disproportionately impacts the lower class. I know that you should first and foremost look at the whether the reasoning is correct , but aside from that was the strength of the conclusion (“ should adopt policies”) a tip off that I should’ve accepted a weaker answer? Can u guys give any general tips on when it’s fine to accept a weaker answer on Strengthen answers?
User avatar
 ashpine17
  • Posts: 321
  • Joined: Apr 06, 2021
|
#89950
What does proportionally mean in the context of the stimulus? Does it mean much greater or just greater?
User avatar
 Beatrice Brown
PowerScore Staff
  • PowerScore Staff
  • Posts: 75
  • Joined: Jun 30, 2021
|
#89967
Hi LT! Thanks for your question, and happy to help :)

Answer choice (C) is not necessarily a weaker answer. What answer choice (C) does to strengthen the conclusion is it defends the conclusion against a possible objection to the argument. To see why, let's break down the argument.

The conclusion of the argument is that democratic societies should adopt policies that ensure constant economic expansion. The reason the author offers to support this conclusion is that economic expansion allows people to improve their economic standing. But what if the rich just get richer and those who are not rich do not really improve their economic standing? If that was the case, then the wide income gap that the author discusses as dangerous may get worse, meaning that democratic societies shouldn't adopt policies that ensure constant economic expansion because it may endanger society further.

Answer choice (C) tells us that economic expansion will actually help those at low income levels more so than those at other income levels. So the concern that we pointed out when examining the argument is not actually a concern! If economic expansion helps those at low income levels most with improving their economic standing, then economic expansion is a good thing for democratic societies because it protects against this class segmentation that has negative effects by preventing wide income gaps.

Therefore, answer choice (C) is the correct answer choice because it defends the argument against a potential weakness. Defending an argument against its weakness is one way to strengthen an argument. This sort of correct answer choice is not necessarily a weaker answer; it just strengthens the conclusion in a different way from an answer choice that actively strengthens the argument by adding a piece of information to support the reasoning. This type of answer choice is similar to Defender Assumption answer choices for Assumption questions; Defender Assumption answer choices aren't weaker than Supporter Assumption answer choices, but they just support the argument in a different way.

I hope this helps, and let me know if you have any other questions!
User avatar
 Beatrice Brown
PowerScore Staff
  • PowerScore Staff
  • Posts: 75
  • Joined: Jun 30, 2021
|
#89968
Hi Ashpine! Happy to help with this :)

In answer choice (C), "proportionally greater increase in earnings for people at low income levels" means that economic expansion will lead to increases in earnings at all income levels, but those at low income levels will have a greater increase in earnings than other economic classes. In other words, everyone's earnings will benefit from economic expansion, but those at low income levels will benefit the most. This helps the argument since it ensures that the income gap won't widen from economic expansion; if those at low income levels have a proportionally greater increase in earnings compared to other groups, then this will help close the income gap even if everyone's earnings increase.

I hope this helps, and please let me know if you have any other questions!

Get the most out of your LSAT Prep Plus subscription.

Analyze and track your performance with our Testing and Analytics Package.